Physics for Scientists and Engineers: A Strategic Approach with Modern Physics (3rd Edition)

Published by Pearson
ISBN 10: 0321740904
ISBN 13: 978-0-32174-090-8

Chapter 25 - Electric Charges and Forces - Exercises and Problems - Page 746: 26

Answer

See the detailed answer below.

Work Step by Step

$$\color{blue}{\bf [a]}$$ We know that the electric field for a positive charge is given by $$\vec E=\dfrac{kq}{r^2} \;\hat r$$ where $\hat r$ is the unit vector in the direction of the electric field. So to find the electric field at the three given points, we need to sketch them as seen below. Hence, $$\vec E_A= \dfrac{kq}{r^2}\;\cos\theta_A \;\hat i+\dfrac{kq}{r^2}\;\sin\theta_A\;\hat j$$ $$\vec E_A= \dfrac{(8.99\times 10^9)(12\times 10^{-9})}{(5\times 10^{-2})^2}\cos0^\circ\;\hat i+\\\dfrac{(8.99\times 10^9)(12\times 10^{-9})}{(5\times 10^{-2})^2}\;\sin0^\circ\;\hat j$$ $$\vec E_A=(\color{red}{\bf4.32\times 10^4}\;{\rm N/c})\hat i$$ $$\vec E_B=\dfrac{kq}{r^2}\;\cos\theta_B \;\hat i+\dfrac{kq}{r^2}\;\sin\theta_B\;\hat j$$ where $\theta_B=180^\circ-\theta$, so $\sin(180^\circ-\theta)=\sin\theta=\dfrac{y}{r}$, and hence, $\cos(180^\circ-\theta)=-\cos\theta=-\dfrac{x}{r}$. Thus, $$\vec E_B=\dfrac{-kq}{r^3} x\;\hat i+\dfrac{ kq}{r^3}y \;\hat j$$ recall that $r=\sqrt{x^2+y^2}$ $$\vec E_B=\dfrac{-kq}{ (x^2+y^2 )^{\frac{3}{2}}} x\;\hat i+\dfrac{ kq}{ (x^2+y^2 )^{\frac{3}{2}}} y \;\hat j$$ Plug the known; $$\vec E_B=\dfrac{-(8.99\times 10^9)(12\times 10^{-9}) (0.05)}{ ( (-0.05)^2+ (0.05)^2 )^{\frac{3}{2}}} \;\hat i+\dfrac{(8.99\times 10^9)(12\times 10^{-9}) (0.05)}{ ( (-0.05)^2+ (0.05)^2 )^{\frac{3}{2}}} \;\hat j$$ $$\vec E_B=(\color{red}{\bf- 1.5\times 10^4}\;{\rm N/c})\hat i+(\color{red}{\bf 1.5\times 10^4}\;{\rm N/c})\hat j$$ $$\vec E_C=\dfrac{kq}{r^2}\;\cos\theta_B \;\hat i+\dfrac{kq}{r^2}\;\sin\theta_B\;\hat j$$ where $\theta_C=180^\circ+\theta$, so $\sin(180^\circ+\theta)=-\sin\theta=-\dfrac{y}{r}$, and hence, $\cos(180^\circ+\theta)=-\cos\theta=-\dfrac{x}{r}$. Thus, $$\vec E_C=\dfrac{-kq}{r^3} x\;\hat i+\dfrac{-kq}{r^3}y \;\hat j$$ recall that $r=\sqrt{x^2+y^2}$ $$\vec E_C=\dfrac{-kq}{ (x^2+y^2 )^{\frac{3}{2}}} x\;\hat i+\dfrac{-kq}{ (x^2+y^2 )^{\frac{3}{2}}} y \;\hat j$$ $$\vec E_C=\dfrac{-kq}{ (x^2+y^2 )^{\frac{3}{2}}} \left[ x\;\hat i+ y \;\hat j\right]$$ Plug the known; $$\vec E_C=\dfrac{-(8.99\times 10^9)(12\times 10^{-9}) }{ ( (-0.05)^2+ (-0.05)^2 )^{\frac{3}{2}}} \left[ (0.05)\;\hat i+ (0.05) \;\hat j\right]$$ $$\vec E_C=(\color{red}{\bf- 1.5\times 10^4}\;{\rm N/c})\hat i+(\color{red}{\bf -1.5\times 10^4}\;{\rm N/c})\hat j$$ --- $$\color{blue}{\bf [b]}$$ See the graph below.
Update this answer!

You can help us out by revising, improving and updating this answer.

Update this answer

After you claim an answer you’ll have 24 hours to send in a draft. An editor will review the submission and either publish your submission or provide feedback.